LSAT and Law School Admissions Forum

Get expert LSAT preparation and law school admissions advice from PowerScore Test Preparation.

 Administrator
PowerScore Staff
  • PowerScore Staff
  • Posts: 8917
  • Joined: Feb 02, 2011
|
#26365
Complete Question Explanation
(See the complete passage discussion here: lsat/viewtopic.php?t=10971)

The correct answer choice is (E)

Thanks to the VIEWSTAMP analysis above, answering this Purpose question should not be particularly challenging.

Answer choice (A): This is the Opposite answer, as the author is the one attacking—not defending—the reform proposed by the LRCWA.

Answer choice (B): This answer choice is incorrect, because the author does not offer a remedy to the problem described in the passage.

Answer choice (C): This may seem like an attractive answer, but it contains an exaggeration. While the author is highly critical of LRCWA’s proposal, she does not claim that it would actually worsen the situation it was intended to improve. If that were true, the author would have to show that the LRCWA’s recommendations would actually enable lawyers to gain disproportionately from awards of damages secured for their clients.

Answer choice (D): This answer choice is incorrect, because the author believes that the LRCWA’s proposals would have a decidedly negative effect on the legal system, harming both lawyers and some of their clients.

Answer choice (E): This is the correct answer choice. The author explains the reform suggested by the LRCWA in the first two paragraphs, and critically evaluates it in the last two paragraphs.

Get the most out of your LSAT Prep Plus subscription.

Analyze and track your performance with our Testing and Analytics Package.